Join Examsbook
225 0

Direction: What approximate value will come in place of the question mark (?) in the following question? (You are not expected to calculate the exact value)

Q:

(1110.02 + 89.81) ÷ ? – $${\sqrt{15}}$$= 15.98

  • 1
    60
  • 2
    120
  • 3
    80
  • 4
    75
  • 5
    40
  • Show AnswerHide Answer
  • Workspace

Answer : 1. "60 "

Are you sure

  Report Error

Please Enter Message
Error Reported Successfully